LSAT and Law School Admissions Forum

Get expert LSAT preparation and law school admissions advice from PowerScore Test Preparation.

User avatar
 Dave Killoran
PowerScore Staff
  • PowerScore Staff
  • Posts: 5850
  • Joined: Mar 25, 2011
|
#94643
Complete Question Explanation
(The complete setup for this game can be found here: lsat/viewtopic.php?f=297&t=1767)

The correct answer choice is (A)

Perhaps because the four Templates make the first five questions so easy to solve, the test makers finish this game by adding an entirely new variable to the game. This variable is added without suspending any of the rules, so all of the previous conditions still hold. But, the addition of a new, unfettered variable (a random, actually) opens up the game to more than four basic orders.

Because there were previously four basic orders, the addition of a new variable creates six additional options for each basic order (one when J is first, another when J is second, and so on). Thus, the introduction of the new variable means there are now 24 possible orders of the cars instead of 4. Thus, the answer to this question is unlikely to be found through re-diagramming the game. Instead, search the answers for a more universal statement that can never be true.

Answer choice (A) is the correct answer. If O is second or third, then O can never receive a premium wash because the second and third cars receive the same type of wash, and there is only one premium wash. If O is fourth—the only other position for O based on the super-sequence—then O will be in front of M and receive a regular wash per the sixth rule. Thus, O can never receive a premium wash, and this answer choice cannot be true and is thus correct.

Answer choices (B) through (E) all could be true, and are therefore incorrect.
 SherryZ
  • Posts: 124
  • Joined: Oct 06, 2013
|
#12017
Thank you for your generous help!

Dec 1999 LSAT Sec 1, Game #3:

I got all the questions correctly EXCEPT Q16 :(

Could you explain why Q16 means? I don't understand :( The question says ADD J in the original stimulus. However, J does not have any limitation, so J can go everywhere, right? I have no idea how to set up this situation :(

Also, could you show me how to set up this game? How many possibilities it has? I wanna make sure that I used the most efficient way to finish this game.

Thanks again!

---Sherry
User avatar
 Dave Killoran
PowerScore Staff
  • PowerScore Staff
  • Posts: 5850
  • Joined: Mar 25, 2011
|
#12018
Hi Sherry,

Yes, J is an entirely new variable that is simply added the preexisting variables presented in the setup.

The best approach to this game is to diagram the four templates for this game. The four templates are based on the sequence which dominates the game:


..... ..... ..... ..... O > M > F
..... ..... ..... V > - - - - - - - - - -
..... ..... ..... ..... ..... T


The templates make solving the questions very easy, so adding a new variable really mixes things up. Because J is a random, there are suddenly many new solutions to the game, and re-diagramming the game doesn't seem like an approach that will be possible under the time constraints.

Thus, whereas you would typically re-diagramming the game to account for the new variable, because that variable is a random you should instead search the answers for a more universal statement that can never be true (because this is a Cannot Be True question).

Answer choice (A) is the correct answer. If O is second or third, then O can never receive a premium wash because the second and third cars receive the same type of wash, and there is only one premium wash. If O is fourth—the only other position for O based on the super-sequence—then O will be in front of M and receive a regular wash per the sixth rule. Thus, O can never receive a premium wash, and this answer choice cannot be true and is thus correct.

Please let me know if that helps. Thanks!
User avatar
 Adam354
  • Posts: 29
  • Joined: Feb 08, 2022
|
#93682
If we put J as the first car washed to test whether O can get a premium wash:

J V O T M F (O has to get the same carwash as V, Rule #3)
J V T O M F (O has to get the same carwash as M, (Rule #6)

Since only one car can get a premium wash, and O gets the same carwash with at least one other car, it cannot receive a premium wash.
 Adam Tyson
PowerScore Staff
  • PowerScore Staff
  • Posts: 5153
  • Joined: Apr 14, 2011
|
#93689
That's correct, Adam, but it's more than just J going first that dictates that answer. No matter where we put that J, O will be unable to get a Premium wash! Testing J in first position does help eliminate some wrong answers, so it's not a bad strategy to start your attack on this question, but it doesn't completely solve the question. Consider your first hypothetical here of JVOTMF: in this case, V and O could both get Super washes, eliminating answer B. Or, VOTM could all get Regular washes in that scenario, eliminating answer C. But answers A, D, and E are all still impossible in that case, so they would all still be contenders. We cannot yet be sure that A is the correct answer based solely on that hypo!

Next, trying your hypo of JVTOMF eliminates none of those remaining contender answers. To prove that A is the only one that absolutely CANNOT be true, we have to do more!

At this point I would try testing the answers more directly, rather than using a hypothetical, and I would probably start with answer A. At this point, Dave's breakdown earlier in this thread really gets the job done efficiently. If O is going to get a Premium wash then it cannot go second or third, so we have to plug it in at 4th. That means the first three variables are J (in any of the first three positions) and V before T, followed by the OMF sequence. Try drawing that as (J, V-T)OMF. In this case, O must get a Regular wash since it is immediately before M. That's it, there is no way that O can have a Premium wash! Answer A is proven to be correct.

Answer E is easy to eliminate because this new J is random. If we were not absolutely convinced that answer A was correct at this point, we would have to test answer D by putting VOM as the first three cars, after which J, T, and F are all interchangeable in the last three spaces. There would be no problem giving them all Super washes, with V getting a Premium wash, so answer D is possible and therefore a wrong answer.

Be careful about your use of hypotheticals to solve questions! It can be time consuming, and just because an answer appears to be correct based on one or two such hypos, that doesn't mean it is correct in ALL solutions.

Get the most out of your LSAT Prep Plus subscription.

Analyze and track your performance with our Testing and Analytics Package.